A geometry problem by Prayas Rautray

Geometry Level pending

Among all triangles with perimeter 21, what is area of the triangle which has the maximum area among all of them?


The answer is 21.21.

This section requires Javascript.
You are seeing this because something didn't load right. We suggest you, (a) try refreshing the page, (b) enabling javascript if it is disabled on your browser and, finally, (c) loading the non-javascript version of this page . We're sorry about the hassle.

1 solution

Marta Reece
Aug 19, 2017

21 / 3 = 7 21/3=7

3 2 × 7 2 21.22 \dfrac{\sqrt3}2\times7^2\approx\boxed{21.22}

How do you know equilateral triangle has the maximum area. Can you give a proof.

Prayas Rautray - 3 years, 9 months ago

Log in to reply

The formula for the area as a function of sides is A = s ( s a ) ( s b ) ( s c ) A=\sqrt{s(s-a)(s-b)(s-c)} , where c = 2 s a b c=2s-a-b .

In terms of a a and b b , this is A = s ( s a ) ( s b ) ( a + b 2 s ) A=\sqrt{s(s-a)(s-b)(a+b-2s)}

The formula is fully symmetrical in a a and b b , so any extreme must exist only at a = b a=b and it can easily be shown to be the maximum by substituting a few values.

Marta Reece - 3 years, 9 months ago

Log in to reply

No, just because it's symmetrical in a a and b b , that doesn't mean that the min/max occurs when a = b a=b .

Take the following counterexample:

a 0 , b 0 , a + b = 10 , maximize ( ( a 5 ) ( b 5 ) ) . a \geq 0 , \quad b \geq 0 , \quad a+b=10 , \quad \text{ maximize } ( \, (a-5)(b-5) \, ).

See the relevant article: Inequalities with strange equality conditions .

Pi Han Goh - 3 years, 9 months ago

Log in to reply

@Pi Han Goh But this is a situation where a a and b b cannot vary independently, as mine do.

Marta Reece - 3 years, 9 months ago

Log in to reply

@Marta Reece What do you mean by "cannot vary independently"?

Pi Han Goh - 3 years, 9 months ago

@Pi Han Goh Thanks. This is really interesting!!!

Prayas Rautray - 3 years, 9 months ago

Log in to reply

@Prayas Rautray Pi Han Goh sir, why don't you post your comment as a solution.

Prayas Rautray - 3 years, 9 months ago

My book gave me the hint that it can be proven by AM-GM inequality. But I couldn't figure it out. Can you help me with this.

Prayas Rautray - 3 years, 9 months ago

Log in to reply

This question is essentially a variation of Weitzenböck's inequality .

Knowing that p = 21 p = 21 and p 2 12 3 T p^2 \geq 12 \sqrt 3 T , where T T represents the area, we can get min ( T ) = 49 3 4 \min(T) = \boxed{\dfrac{49\sqrt3}{4}} .

Pi Han Goh - 3 years, 9 months ago

0 pending reports

×

Problem Loading...

Note Loading...

Set Loading...